Difference between revisions of "2005 AMC 12B Problems/Problem 6"
(problem, solution, and category!) |
5849206328x (talk | contribs) m (Fixed heading) |
||
Line 1: | Line 1: | ||
− | + | == Problem == | |
In <math>\triangle ABC</math>, we have <math>AC=BC=7</math> and <math>AB=2</math>. Suppose that <math>D</math> is a point on line <math>AB</math> such that <math>B</math> lies between <math>A</math> and <math>D</math> and <math>CD=8</math>. What is <math>BD</math>? | In <math>\triangle ABC</math>, we have <math>AC=BC=7</math> and <math>AB=2</math>. Suppose that <math>D</math> is a point on line <math>AB</math> such that <math>B</math> lies between <math>A</math> and <math>D</math> and <math>CD=8</math>. What is <math>BD</math>? | ||
== Solution == | == Solution == |
Revision as of 17:26, 23 April 2009
Problem
In , we have and . Suppose that is a point on line such that lies between and and . What is ?
Solution
Draw height . We have that . From the Pythagorean Theorem, . Since , , and , so .